What are the absolute extrema of the given function on the given set?

Click For Summary
The discussion focuses on finding the absolute extrema of the function f(x,y) = x^2 + 4y^2 + 3x - 1 within the circular domain defined by x^2 + y^2 ≤ 4. A critical point was identified at (-3/2, 0), yielding a function value of -1, but there was confusion regarding the calculation. Participants emphasized the importance of evaluating the function on the boundary of the circle by substituting y = √(4 - x^2) into f(x,y) and finding the corresponding values. Ultimately, the correct approach involves taking the derivative of the modified function to locate additional critical points and determine the absolute extrema.
Reefy
Messages
62
Reaction score
1

Homework Statement



Find the absolute extrema of the function on the set D.

f(x,y)= x^2 + 4y^2 + 3x -1

D= {(x,y) l x^2 + y^2 ≤ 4}

Homework Equations





The Attempt at a Solution



The only thing I've done so far was find the critical point. I found f-sub x = 2x+3 and f-suby= 8y giving me a critical point of (-3/2,0), correct?

Using that critical point, I found f(-3/2,0) = -1

After that, I don't know how to proceed. I know -2 ≤ x ≤ 2 and -2 ≤ y ≤ 2 but how do I find the other critical numbers with that info? I'm unsure because the closed boundary graph is a circle and not a square, box, or triangular region.
 
Physics news on Phys.org
Reefy said:

Homework Statement



Find the absolute extrema of the function on the set D.

f(x,y)= x^2 + 4y^2 + 3x -1

D= {(x,y) l x^2 + y^2 ≤ 4}

Homework Equations


The Attempt at a Solution



The only thing I've done so far was find the critical point. I found f-sub x = 2x+3 and f-suby= 8y giving me a critical point of (-3/2,0), correct?

Using that critical point, I found f(-3/2,0) = -1

f(-3/2) is not equal to -1.

Reefy said:
After that, I don't know how to proceed. I know -2 ≤ x ≤ 2 and -2 ≤ y ≤ 2 but how do I find the other critical numbers with that info? I'm unsure because the closed boundary graph is a circle and not a square, box, or triangular region.

The absolute extreme can be on the perimeter. Find the critical point with the condition that x2+y2=4.

ehild
 
ehild said:
f(-3/2) is not equal to -1.

Ah, you're right, thank you. For that, I got -13/4.

ehild said:
The absolute extreme can be on the perimeter. Find the critical point with the condition that x2+y2=4.

ehild

So set y=√(4-x^2) ? and find f(x,√(4-x^2)). Then find values of x and use the result to find y. Then find critical numbers? Because that's what I did but it didn't seem right.
 
Last edited:
Reefy said:

Homework Statement



Find the absolute extrema of the function on the set D.

f(x,y)= x^2 + 4y^2 + 3x -1

D= {(x,y) l x^2 + y^2 ≤ 4}

Homework Equations





The Attempt at a Solution



The only thing I've done so far was find the critical point. I found f-sub x = 2x+3 and f-suby= 8y giving me a critical point of (-3/2,0), correct?

Using that critical point, I found f(-3/2,0) = -1

After that, I don't know how to proceed. I know -2 ≤ x ≤ 2 and -2 ≤ y ≤ 2 but how do I find the other critical numbers with that info? I'm unsure because the closed boundary graph is a circle and not a square, box, or triangular region.

You have found the global minimum of f, subject to g = x^2 + y^2 <= 4. (although your f is wrong while your (x,y) is correct). However, an extremum can also be a maximum, and you have not yet found that.
 
Reefy said:
Ah, you're right, thank you. For that, I got -13/4.
That is right.

Reefy said:
So set y=√(4-x^2) ? and find f(x,√(4-x^2)). Then find values of x and use the result to find y. Then find critical numbers? Because that's what I did but it didn't seem right.

Show your work.

ehild
 
Thanks, guys! I got it now. I forgot to take the derivative of f(x,√(4-x^2)).

Instead of finding the derivative, I found x and y of f(x,√(4-x^2)) and plugged it into the original function which was wrong.
 
Question: A clock's minute hand has length 4 and its hour hand has length 3. What is the distance between the tips at the moment when it is increasing most rapidly?(Putnam Exam Question) Answer: Making assumption that both the hands moves at constant angular velocities, the answer is ## \sqrt{7} .## But don't you think this assumption is somewhat doubtful and wrong?

Similar threads

  • · Replies 11 ·
Replies
11
Views
2K
  • · Replies 6 ·
Replies
6
Views
2K
  • · Replies 7 ·
Replies
7
Views
3K
  • · Replies 8 ·
Replies
8
Views
2K
  • · Replies 11 ·
Replies
11
Views
2K
Replies
11
Views
2K
  • · Replies 4 ·
Replies
4
Views
2K
  • · Replies 1 ·
Replies
1
Views
2K
Replies
25
Views
3K
  • · Replies 20 ·
Replies
20
Views
1K